K
Khách

Hãy nhập câu hỏi của bạn vào đây, nếu là tài khoản VIP, bạn sẽ được ưu tiên trả lời.

20 tháng 10 2019

Chọn A.

Ta có nên đồ thị hàm số có tiệm cận đứng là x = -2

Ta có  nên đồ thị hàm số có tiệm cận ngang là y = -3.

20 tháng 11 2018

24 tháng 7 2018

Đáp án C

AH
Akai Haruma
Giáo viên
29 tháng 5 2021

Lời giải:

TXĐ: \((-\infty; -1)\cup (-1;+\infty)\)
\(\lim\limits_{x\to +\infty}y=\lim\limits_{x\to +\infty}\frac{1+\sqrt{1+\frac{1}{x}}}{1+\frac{1}{x}}=\frac{1+1}{1}=2\)

\(\lim\limits_{x\to -\infty}y=\lim\limits_{x\to -\infty}\frac{-1+\sqrt{1+\frac{1}{x^2}}}{-1+\frac{1}{-x}}=\frac{-1+1}{-1}=0\)

Do đó ĐTHS có 2 TCN là $y=0$ và $y=2$

\(\lim\limits_{x\to -1-}y=\lim\limits_{x\to -1-}\frac{x+\sqrt{x^2+1}}{x+1}=-\infty\) do \(\lim\limits_{x\to -1-}(x+\sqrt{x^2+1})=\sqrt{2}-1>0\) và \(\lim\limits_{x\to -1-}\frac{1}{x+1}=-\infty\)

Tương tự \(\lim\limits_{x\to -1+}y=+\infty\) nên $x=-1$ là TCĐ của đths

Vậy có tổng 3 TCN và TCĐ

 

11 tháng 11 2017

5 tháng 6 2019

Đáp án A

lim x → - 1 + x - 1 x + 1 = - ∞

lim x → - 1 - x - 1 x + 1 = + ∞

Vậy hàm số có đường tiệm cận đứng là x = -1

 

NV
2 tháng 9 2021

\(\lim\limits_{x\rightarrow+\infty}\dfrac{\sqrt{x-1}}{x^2-3x+2}=\lim\limits_{x\rightarrow+\infty}\dfrac{\sqrt{\dfrac{1}{x^3}-\dfrac{1}{x^4}}}{1-\dfrac{3}{x}+\dfrac{2}{x^2}}=0\)

\(\Rightarrow y=0\) là tiệm cận ngang

\(\lim\limits_{x\rightarrow1^+}\dfrac{\sqrt{x-1}}{x^2-3x+2}=\lim\limits_{x\rightarrow1^+}\dfrac{1}{\sqrt{x-1}\left(x-2\right)}=\infty\)

\(\Rightarrow x=1\) là tiệm cận đứng

\(\lim\limits_{x\rightarrow2}\dfrac{\sqrt{x-1}}{x^2-3x+2}=\dfrac{1}{0}=\infty\)

\(\Rightarrow x=2\) là tiệm cận đứng

ĐTHS có 1 TCN và 2 TCĐ

17 tháng 8 2019

20 tháng 1 2019

Chọn A

Đk để hàm số xác định là: . Vậy mệnh đề đúng.

Do hàm số có tập xác định nên không tồn tại do đó đồ thị hàm số này không có đường tiệm cận ngang. Vậy mệnh đề sai.

Do nên đồ thị hàm số có đường tiệm cận đứng là . Vậy đúng.

Ta có

Do bị đổi dấu qua nên hàm số có một cực trị. Vậy mệnh đề đúng.

 

Do đó số mệnh đề đúng là .